1. The following data show weight (in kg) of 24 women in a study: 46. 4, 53. 2, 52. 8, 42. 0, 50. 8,
43. 0, 51. 9, 59. 2, 55. 1, 38. 9, 49. 7, 49. 9, 43. 1,42. 2, 52. 7. 49. 8. 50. 7, 44. 8. 49. 2, 47. 7, 42. 9,
52. 9, 54. 1, 45. 4.
Prepare the following:
I.
Calculate a) mean, b) median, c) mode, d) variance, e) standard deviation, f)
coefficient variation, g) IQR
Box and whisker plot
II.
III.
Discuss the distribution of these data​

Answers

Answer 1

The mean is 48.47 kg, median is 49.55 kg, mode is not available, variance is 34.1 kg², standard deviation is 5.84 kg, coefficient of variation is 12.03% and IQR is 8.35 kg.

The given data shows the weight (in kg) of 24 women in a study. To analyze the data, we need to calculate various statistical measures:

I. Statistical Measures:
a) Mean = (Sum of all weights) / (Number of observations) = (1163.4) / (24) = 48.47 kg
b) Median = Middle value of the sorted data set = 49.55 kg
c) Mode = The most frequent value in the data set = No mode as there are no repeating values.
d) Variance = (Sum of squares of deviations of each value from mean) / (Number of observations) = 34.1 kg²
e) Standard deviation = Square root of variance = 5.84 kg
f) Coefficient of variation = (Standard deviation / Mean) x 100 = 12.03%
g) IQR (Interquartile range) = Q3 - Q1 = 53.025 - 44.675 = 8.35 kg

II. Box and Whisker Plot:
The box and whisker plot displays the distribution of the data. The lower and upper quartiles are represented by the bottom and top of the box respectively, and the median is represented by the line in the middle. The whiskers represent the minimum and maximum values.

III. Distribution:
The data set appears to be skewed to the right as the median is less than the mean. There are no outliers in the data, and the IQR is relatively small, indicating that the data is not too spread out. The coefficient of variation is moderate, indicating that the data has a moderate degree of variation. Overall, the data set seems to be fairly normal, with a few outliers on the right side.

Know more about mean here:

https://brainly.com/question/31101410

#SPJ11


Related Questions

KAP


1 IN PU


1. Luis parents give him x dollars for his monthly allowance. Each month, he must


pay $35 for his cell phone. One-sixth of the remaining money can be spent on


entertainment. Which function can be used to find the amount in dollars Luis can


spend on entertainment?


A f(x) =*735


B. F(x) = 35x - 3


C. F(x) = 35 - $


D. F(x) = § - 35

Answers

The function can be used to find the amount in dollars Luis can spend on entertainment is 1/6(x-35)

Total amount of money luis get from his parent = x dollars

The amount luis has to pay to his parents for his cell phone is $35

After giving money for cell phone the amount of money left with luis will be x - 35

so, remaining money with luis = x - 35

One sixth of the remaining money which can be spent on entertainment by luis is 1/6(x-35)

The function can be written in the form of

f(x) = 1/6(x-35)

To know more about function click here :

https://brainly.com/question/12431044

#SPJ4

The area of the triangle below is \frac{2}{25}

25

2



square feet. What is the length of the base? Express your answer as a fraction in simplest form.

1/5 f

Answers

The length of the base of the given triangle can be simplified as 2√2/5 feet, which is equivalent to √8/5 feet.

What is the length of the base of a triangle if its area is (2/25) * 252 square feet and the height is twice the length of the base?

We are given that the area of the triangle is (2/25) * 252 square feet.

Let the length of the base be x. Then, the height of the triangle can be expressed as (2/5)x, since the base divides the triangle into two equal parts.

The area of the triangle is given by the formula A = (1/2)bh, where b is the length of the base and h is the height of the triangle.

Substituting the given values, we get:

(1/2)x(2/5)x = (2/25)*252

Simplifying this equation, we get:

(1/5)x²= 20.16

Multiplying both sides by 5, we get:

x² = 100.8

Taking the square root of both sides, we get:

x =√(100.8)

Simplifying this expression, we get:

x = √(25*4.032)x = 5*√(4.032)x = (5/5)*√(4.032)x = 1*√(4.032)

Therefore, the length of the base is √(4.032) feet, which can be expressed as a fraction in simplest form as 2√(2)/5 feet.

Learn more about triangle

brainly.com/question/2773823

#SPJ11

What is 3x-(2x+9) + 4x?
Please help^^

Answers

Answer:

5x-9

Step-by-step explanation:

Distribute: 3x-(2x+9) + 4x

3x - 2x - 9 + 4x

Combine Like Terms: 3x - 2x - 9 + 4x

5x-9

3x-(2x+9)+4x
first distribute the negative
3x-2x-9+4x
combine like terms
5x-9

Shandra has $760 to spend at a bicycle store for some new gear and biking outfits. Assume all prices listed include tax. ⢠She buys a new bicycle for $433. 54. ⢠She buys 2 bicycle reflectors for $18. 41 each and a pair of bike gloves for $10. 76. ⢠She plans to spend some or all of the money she has left to buy new biking outfits for $66. 40 each. Write and solve an inequality which can be used to determine o, the number of outfits Shandra can purchase while staying within her budget. â

Answers

We can use an inequality to determine the number of biking outfits Shandra can purchase while staying within her budget.

Let o represent the number of outfits she can purchase. Here are the given terms and costs:
- Initial budget: $760
- Bicycle cost: $433.54
- 2 reflectors cost: 2 * $18.41 = $36.82
- Bike gloves cost: $10.76
- Outfit cost: $66.40 each

Now, we can set up the inequality:
760 >= 433.54 + 36.82 + 10.76 + 66.40 * o

First, combine the constants:
760 >= 481.12 + 66.40 * o

Now, subtract 481.12 from both sides:
278.88 >= 66.40 * o

Finally, divide both sides by 66.40:
o <= 4.2

Since Shandra can only purchase whole outfits, the maximum number of outfits she can buy is 4. So the inequality representing this situation is o <= 4.

To know more about inequality refer here:

https://brainly.com/question/30231190?#

SPJ11

The temperature at a point (x, y, z) is given byT(x, y, z) = 10e¯2x² − y² − 3z².In which direction does the temperature increase fastest at the point (1, 3, 1)?Express your answer as a UNIT vector.

Answers

The direction of fastest increase in temperature at the point (1, 3, 1) is given by the unit vector (-5/sqrt(10) , -3/sqrt(10) , -9/sqrt(10)).

To find the direction of fastest increase in temperature at the point (1, 3, 1), we need to find the gradient of the temperature function T(x, y, z) at that point.

The gradient of a function is a vector that points in the direction of steepest increase, and its magnitude is the rate of change in that direction. So, we can find the gradient vector ∇T(x, y, z) as follows:

∇T(x, y, z) = ( ∂T/∂x , ∂T/∂y , ∂T/∂z )

=[tex]( -20xe^(-2x^2-y^2-3z^2) , -2ye^(-2x^2-y^2-3z^2) , -6ze^(-2x^2-y^2-3z^2) )[/tex]

Therefore, at the point (1, 3, 1), the gradient of T(x, y, z) is:

∇T(1, 3, 1) = [tex]( -20e^(-8) , -6e^(-8) , -18e^(-8) )[/tex]

To find the direction of fastest increase, we need to normalize this vector to a unit vector. The magnitude of the gradient vector is:

|∇T(1, 3, 1)| = sqrt( (-[tex]20e^(-8))^2 + (-6e^(-8))^2 + (-18e^(-8))^2 )[/tex]

= sqrt( 640e^(-16) )

= 8e^(-8) sqrt(10)

So, the unit vector in the direction of fastest increase is:

( -20e^(-8) / (8e^(-8) sqrt(10)) , -6e^(-8) / (8e^(-8) sqrt(10)) , -18e^(-8) / (8e^(-8) sqrt(10)) )

= ( -5/sqrt(10) , -3/sqrt(10) , -9/sqrt(10) )

Therefore, the direction of fastest increase in temperature at the point (1, 3, 1) is given by the unit vector (-5/sqrt(10) , -3/sqrt(10) , -9/sqrt(10)).

To learn more about unit vector visit: https://brainly.com/question/30279109

#SPJ11

Hallar la altura de una asta bandera, si un estudiante la observa desde un punto a, con un ángulo de 30° y entre el estudiante y la asta hay una distancia de 10m.

Answers

Answer:

The height of the flagpole is approximately 5.774 meters.

Step-by-step explanation:

Let's call the height of the flagpole h. We can use trigonometry to set up the following equation:

tan(30°) = h/10

Simplifying this equation, we get:

h = 10 tan(30°)

Using a calculator, we find that tan(30°) ≈ 0.5774, so:

h ≈ 5.774 meters

Therefore, the height of the flagpole is approximately 5.774 meters.

To know more about flagpole refer here

https://brainly.com/question/31296662#

#SPJ11

Mai drew the design shown below. Each
rectangle in the design has the same
area. Each rectangle is what fraction of
the area of the complete design?

Answers

Each rectangle is 1/3 of the area of the complete design.

What fraction of the area of the complete design?

A fraction represents the parts of a whole or collection of objects e.g. 3/4 shows that out of 4 equal parts, we are referring to 3 parts.

Looking at the design, you will be notice that the main (bigger) rectangle is divided to three smaller rectangles. Thus, each rectangle is one out of three  rectangles i.e. 1/3.

Therefore, each rectangle is 1/3 of the area of the complete design.

Learn more about fraction on:

brainly.com/question/11562149

#SPJ1

Complete Question

Check attached image

Need this fast
A) -24 Solve lim 69²-24 B) 4 a+2 2-a C) 24 D) - 4

Answers

The limit of (69²-24) as x approaches infinity is equal to infinity.

As x approaches infinity, the value of (69²-24) becomes very large, and it goes to infinity. Therefore, the limit of (69²-24) as x approaches infinity is infinity.

B) The limit of (4a+2)/(2-a) as a approaches 2 from the left is equal to -6 and as a approaches 2 from the right is equal to 6.

As a approaches 2 from the left, the denominator (2-a) approaches zero from the negative side, and the numerator (4a+2) approaches -6. Therefore, the limit of (4a+2)/(2-a) as a approaches 2 from the left is -6.

As a approaches 2 from the right, the denominator (2-a) approaches zero from the positive side, and the numerator (4a+2) approaches 6. Therefore, the limit of (4a+2)/(2-a) as a approaches 2 from the right is 6.

For more questions like Limit click the link below:

https://brainly.com/question/12211820

#SPJ11

callum says 300cm2 is the same as 3m2 because there are 100cm in 1m so you divide by 100 callums method is wrong explain why

Answers

Answer:

Callum’s method is incorrect because he is confusing the conversion of linear units with the conversion of square units. There are indeed 100 cm in 1 m, but when converting square units, you need to square the conversion factor. So 1 m² is equal to (100 cm)² or 10,000 cm². Therefore, 300 cm² is equal to 0.03 m², not 3 m².

Step-by-step explanation:

Answer:

The answer is wrong because the 3m is still squared if you divided it by 100 then it should only be 3m not 3m^2.

Step-by-step explanation:

I am not 100% sure this is correct so please dont get mad at me.

Aleks and Melanie used a protractor to measure the angle below. Aleks thinks the angle measures 50° but Melanie says it is actually 130°. Their teacher confirms that Melanie has the correct answer. What mistake did Aleks make while measuring the angle?

Answers

The mistake, Aleks made, while measuring the angle is, he measure the angle from the wrong side of the line.

Angle is a dimensionless vector quantity, that is, it is very important, to take care of the directions, while measuring the angle.

That is, to measure the angle, say ∠ABC, the 0°(reference) line of the protractor, must be on one either AB or BC, to measure the angle rightly.

And since the angles between two lines are supplementary in nature, that is, the two angles will add up to make 180°, that is why, the angle measure by Alek and Melanie, add up to make 180°.

Read more about angle at:

https://brainly.com/question/1309590

What's 2x = 20 if 4y = 80

Answers

Answer:

x=10 and y=20, 2x=20=y

Step-by-step explanation:

2x=20 | Divide by 2 on both sides

x=10

4y=80 | Divide by 4 on both sides

y=20

Answer: 40

80 ÷ 2 = 20, so x = 40.

To check your answer;

40 x 2 = 80

The angle of depression from the top of a 150m high cliff to a boat at sea is 7°. How much closer to the cliff must the boat move for the angle of depression to become 19°?

Answers

The boat must move 785.82 m closer to the cliff  for the angle of depression to become 19°.

We need to find how much closer to the cliff the boat must move for the angle of depression to change from 7° to 19°.

Calculate the distance from the boat to the base of the cliff at 7° angle of depression.
Using the tangent function, we have:
tan(angle) = height/distance
tan(7°) = 150m/distance
distance = 150m/tan(7°)

distance=1221.49

Calculate the distance from the boat to the base of the cliff at 19° angle of depression.
Using the tangent function, we have:
tan(angle) = height/distance
tan(19°) = 150m/distance
distance = 150m/tan(19°)

distance=435.6665

Calculate the difference between the two distances to find out how much closer the boat must move.
difference = distance at 7° angle of depression - distance at 19° angle of depression

Plugging in the values from Steps 1 and 2, we get:
difference = (150m/tan(7°)) - (150m/tan(19°))

difference=1221.49-435.6665

difference=785.8235



After calculating, we find that the boat must move approximately 785.82 meters closer to the cliff for the angle of depression to change from 7° to 19°.

To learn more about angle of depression refer here

https://brainly.com/question/13514202#

#SPJ11

PLEASE HELP (40 POINTS)

Answers

The coordinates of the points N and L are N = (-2d, 0) and L = (-4f, g)

Calculating the coordinates of N and L

From the question, we have the following parameters that can be used in our computation:

M = (-2d - 4f, g)

O = (0, 0)

ON = 2d

Given that

ON = 2d

Then it means that

N = (-2d, 0)

For the point L, we have

LO = MN

Where

LO = √[(x - 0)² + (y - 0)²] i.e. the distance formula

LO = √[x² + y²]

Next, we have

MN = √[(-2d - 4f + 2d)² + (g - 0)²] i.e. the distance formula

MN = √[(-4f)² + g²]

So, we have

LO = MN

√[x² + y²] = √[(-4f)² + g²]

By comparison, we have

x = -4f and y = g

This means that the coordinates of point L = (-4f, g)

Read more about distance at

https://brainly.com/question/28551043

#SPJ1

fast pls
in When calculating 4-√x+15 what option you get x?-1 lim X>1 1 the process -1 A) lim (x + 1) (4 + V1 +15) -1 B) lim 21 (1+1) (4 - V1-15) C) lim 16 - 2 - 1)(4+r+15) D) lím 16-1 (12 - 1) (4 - Vr+15)

Answers

None of the options are correct, and the value of x is simply 1.

Start with the given expression: 4 - √x + 15

Substitute x with the limit value of 1: 4 - √1 + 15 = 18

Therefore, the limit of the given expression as x approaches 1 from the right is 18.

To verify this result using the provided options, we can simplify each option and check which one equals 18 as x approaches 1 from the right.

Option A simplifies to (2/√2) + 2, which equals √2 + 2, not equal to 18.

Option B simplifies to 2(4 - √15), which equals 2(4 - 3.87), approximately equal to 2.27, not equal to 18.

Option C simplifies to 3(4 + √15), which equals 3(4 + 3.87), approximately equal to 23.61, not equal to 18.

Option D simplifies to 3(3)(3), which equals 27, not equal to 18.

Therefore, none of the options are correct, and the value of x is simply 1.

none of the options are correct, and the value of x is simply 1.

For more questions like Limit click the link below:

https://brainly.com/question/12207539

#SPJ11

16. Justin is joining a gym. The gym is currently offering a discount on the fee to join and on the monthly rate.
The discounted price,in dollars,the gym charges can be represented by the equation y=10x+5
a. What are the slope and the Y-intercept of the equation? What do the slope and the Y-intercept each represent in this equation?

Answers

Answer:

The equation y = 10x + 5 is in slope-intercept form, y = mx + b, where m is the slope and b is the y-intercept.

The equation y = 10x + 5 is in slope-intercept form, y = mx + b, where m is the slope and b is the y-intercept.Slope (m) = 10

The equation y = 10x + 5 is in slope-intercept form, y = mx + b, where m is the slope and b is the y-intercept.Slope (m) = 10Y-intercept (b) = 5

The equation y = 10x + 5 is in slope-intercept form, y = mx + b, where m is the slope and b is the y-intercept.Slope (m) = 10Y-intercept (b) = 5The slope represents the rate of change of the monthly rate with respect to the fee to join. For every increase of $1 in the fee to join, the monthly rate increases by $10.

The equation y = 10x + 5 is in slope-intercept form, y = mx + b, where m is the slope and b is the y-intercept.Slope (m) = 10Y-intercept (b) = 5The slope represents the rate of change of the monthly rate with respect to the fee to join. For every increase of $1 in the fee to join, the monthly rate increases by $10.The y-intercept represents the initial cost of joining the gym. It is the amount that the gym charges even if the fee to join is $0. In this case, the gym charges $5 to join.

The slope is 10 which represents the monthly rate. The Y-intercept is 5 which represents the joining fee.

1)Change point A in the scatterplot to point (1,12). Calculate the correlation coefficient and note how much it differs from .96. (2)Change point A back to (1,2) and change point B to (4,15). Calculate the correlation coefficient and note how much it differs from .96. Did the correlation coefficient change more when the point you raised 10 units was in the middle of the scatterplot or at the edge of the scatterplot? Why do you think this is so? (3)Move only one point and make the correlation coefficient become negative. Write about what you did and why it made the correlation go negative.(4) Suppose you had a scatterplot with only two points. Assuming your two points don't define either a horizontal line (both y-values the same) or a vertical line (both x-values the same), what is the correlation coefficient? Why do you think this is true? What happens as you try different points (again, without defining a horizontal or vertical line)?(5)Enter the points (1,2) and (3,2) — this defines a horizontal line. Try to calculate the correlation coefficient. What did your graphing calculator tell you? What happened?(6) Enter the points (1,2) and (1,3) — this defines a vertical line. Try to calculate the correlation coefficient. What did your graphing calculator tell you? What happened? The following scatterplot was constructed by reversing the x- and y-values in the original scatterplot. Without calculating the new correlation coefficient, what do you think r is? Why? (7)Graph depicts 16 scatter plots on a coordinate plane without coordinate points. 7 scatter plots in quadrant 3, 1 scatter plot in quadrant 4, and 8 scatter plots in quadrant 1. The following scatterplot was constructed by taking the negative of each x-value in the original scatterplot. Without calculating the new correlation coefficient, what do you think r is? Why? What would the correlation coefficient be if we took the negative of all the x-values and all the y-values? Graph depicts 15 scatter plots on a coordinate plane without coordinate points. 7 scatter plots in quadra

Answers

The new regression coefficient is about 0.663, viz. lesser than the previous regression coefficient by 0.297. Thus, a single outlier creates a significant drop in the correlation

How to solve

Changing A to (1,12) gives below scatterplot and regression parameters

(check image)

2. In this case, r is about 0.766, a drop of 0.194 which is substantial, but lower than the previous drop. The regression coefficient changed much more when the outlier was in the middle of the scatterplot. This happens because the data series itself is increasing.

So the effect of 10 points in a middle point is much more of an outlier compared to when this 10-point increase happens for the highest value of x. Hence, the r value drops more in the former case.

3. r can become negative if drop the point B to a highly negative y-value. Consider taking it to (4, -50). Then we get the following regression parameters

We obtain r = -0.275. Since the expected y-value was highest for point B, so changing it drastically to a large negative value leads to a negative correlation between the two variables.

4. With only two points that are parallel to neither of the axes, the correlation coefficient is always exactly either 1 or -1. The correlation is 1 if the slope of the line joining the two points is positive, and -1 if the slope is negative.

That is, there is always either a perfect positive correlation or a perfect negative correlation. This is so because there is always a unique line joining two points, which leads to a perfect correlation between them. Even by differing the pairs, this relation shall always hold true.

5. If the points are parallel to the X axis, we should obtain r=0, because it indicates no relation between the variables. So points (1, 2) and (3, 2) lead to r=0. This can be verified using any calculator.

A vertical line also leads to r=0. Since the y value does not change, so no correlation can be established. Actually, it is just like flipping the x and y variables, and we know flipping does not change the correlation coefficient. So we should obtain r=0 even for a vertical line.

Read more about regression here:

https://brainly.com/question/17004137

#SPJ1

Bob and two friends each were able to juggle with bean bags for 3/4 of a minute. How long did they juggle together? No decimals pls!

Answers

Answer:

Step-by-step explanation:

They each juggled for 3/4 of a minute

There were 3 people in total

3 people times 3/4 of a minute equals 2 1/4 minutes

find the general solution of the following linear system. y′ = [2 2 −4 2 −1 −2 4 2 −6] y with λ = −1,−2,−2

Answers

To find the general solution of the linear system y' = [2 2 -4; 2 -1 -2; 4 2 -6] y, we need to first find the eigenvectors and eigenvalues of the coefficient matrix A = [2 2 -4; 2 -1 -2; 4 2 -6].

Using the characteristic equation, we can find the eigenvalues:

det(A - λI) = 0

=> det([2-λ 2 -4; 2 -1-λ -2; 4 2 -6-λ]) = 0

=> (2-λ)[(-1-λ)(-6-λ) - 4] - 2[(-2)(-6-λ) - 8] + 4[2(-1-λ) - 4] = 0

=> λ^3 -  + 8λ - 4 = 0

=> (λ-1)(λ-2[tex])^2[/tex] = 0

Thus, λ = 1, 2 (with multiplicity 2). For each eigenvalue, we need to find a corresponding eigenvector.

For λ = 1, we need to find the null space of the matrix (A - λI):

A - λI = [1 2 -4; 2 -2 -2; 4 2 -7]

=> R2 <- R2 - 2R1, R3 <- R3 - 4R1

[1 2 -4; 0 -6 6; 0 -6 9]

=> R3 <- R3 - R2

[1 2 -4; 0 -6 6; 0 0 3]

So, we have a basic eigenvector of the form [4,-2,1]^T. To obtain a linearly independent eigenvector, we use the method of generalized eigenvectors. We need to find a vector v such that (A - λI) v = u, where u is the basic eigenvector.

(A - λI) v = u

=> [1 2 -4; 2 -2 -2; 4 2 -7] v = [4; -2; 1]

=> R2 <- R2 - 2R1, R3 <- R3 - 4R1

[1 2 -4; 0 -6 6; 0 -6 9] v = [4; -2; 1]

=> R3 <- R3 - R2

[1 2 -4; 0 -6 6; 0 0 3] v = [4; -2; 1]

=> -6v2 + 6v3 = -2

=> 3v3 = 1

=> 2v2 - 4v3 = -2

=> v2 = 0

So, we have v = [0; 1/3; 2/3[tex]]^T[/tex] as the second eigenvector corresponding to λ = 1.

For λ = 2, we need to find the null space of the matrix (A - λI):

A - λI = [0 2 -4; 2 -3 -2; 4 2 -8]

=> R1 <-> R2

[2 -3 -2; 0 2 -4; 4 2 -8]

=> R3 <- R3 - 2R1

[2 -3 -2; 0 2 -4; 0 8 -12]

Learn more about general solution,

https://brainly.com/question/12641320

#SPJ4

Math geometry question please help

Answers

Answer:

LP is a raym∠LMO = 90°m∠LPO = 90°m∠MLP = 26°arc MP = 154°arc MNP = 206°

Step-by-step explanation:

You want various angle and arc measures in the given figure.

Relationships

The relevant angle relationships are ...

a radius to a point of tangency makes a right angle with the tangentan arc has the same measure as its central anglethe sum of the arcs of a circle is 360°the sum of angles in a quadrilateral is 360°an angle is formed from two rays whose endpoints are the vertex of the angle

These answer the given questions as follows:

  LP is a ray.

  The angle at M is 90°.

  The angle at P is 90°.

 The angle at L is 360° -90° -90° -154° = 26°

  Arc MP has the same measure as angle MOP, 154°

  Arc MNP completes the circle, so is 360° -154° = 206°

The prism shown has a surface area of 1,500 mm squared. What is the height, h, of the prism?

Answers

The height of the triangular base prism is 10 mm.

How to find the surface area of a prism?

The prism above is a triangular base prism. The surface area of the prism can be calculated as follows:

surface area of the triangular prism = (a + b + c)l + bh

where

a, b, and c are the side of the triangular basel = height of the prismb = base of the triangleh = height of the triangle

Therefore,

surface area of the triangular prism = (20 + 30 + 40) + 20 × 30

1500 = 90l + 600

1500 - 600 = 90l

90l = 900

divide both sides by 90

l = 900 / 90

l = 10 mm

Therefore,

height of the prism = 10 mm

Learn more on surface area here: https://brainly.com/question/31223288

#SPJ1

The profit ( in hundreds of dollars) from selling units of a product is given by
the profit function P(x) = x^2/ 2x + 1 Find the marginal profit when 4 units are produced
and sold and interpret your answer using words, numbers and units. Be very specific.
(Round the number part of your answer to the nearest cent.)

Answers

the marginal profit when 4 units are produced and sold is approximately -$69.14. This means that when producing and selling the 4th unit, the profit will decrease by $69.14.

To find the marginal profit when 4 units are produced and sold, we first need to find the derivative of the profit function P(x) with respect to x. The given profit function is:

P(x) = (x^2) / (2x + 1)

Now, let's find its derivative, which represents the marginal profit function:

dP/dx = (d/dx(x^2))/(2x + 1) - (x^2)(d/dx(2x + 1))/((2x + 1)^2)

First, find the derivative of x^2 and 2x + 1:

d/dx(x^2) = 2x
d/dx(2x + 1) = 2

Now, substitute these values into the marginal profit function:

dP/dx = (2x)/(2x + 1) - (x^2)(2)/((2x + 1)^2)

Next, we'll find the marginal profit when 4 units are produced and sold. So, let's substitute x = 4 into the marginal profit function:

dP/dx(4) = (2 * 4)/(2 * 4 + 1) - (4^2)(2)/((2 * 4 + 1)^2)

dP/dx(4) = (8)/(9) - (32)(2)/(81)

dP/dx(4) = (8 - 64)/(81)

dP/dx(4) = -56/81

Since the profit is in hundreds of dollars, we need to multiply the marginal profit by 100 to get the value in dollars. Then, round to the nearest cent:

Marginal profit in dollars = (-56/81) * 100 ≈ -$69.14

So, the marginal profit when 4 units are produced and sold is approximately -$69.14. This means that when producing and selling the 4th unit, the profit will decrease by $69.14.

to know more about marginal profit click here:

https://brainly.com/question/29785280

#SPJ11

Penny decided to travel to Palawan. The airplane flew at an average rate of 300 miles per hour and covered 1500 miles. How long will the flight will take? *

A. 3 hours

B. 4 hours

C. 5 hours

D. 6 hours

Answers

The time it will take the flight is C) 5 hours.

To solve this problem, we can use the formula: distance = rate x time. In this case, we know that the distance is 1500 miles and the rate (or speed) is 300 miles per hour. We can rearrange the formula to solve for time: time = distance / rate. Plugging in the values we have, we get:

time = 1500 miles / 300 miles per hour
time = 5 hours

Therefore, the correct answer is C. It will take Penny 5 hours to fly from her starting point to Palawan at an average speed of 300 miles per hour. This calculation assumes that the plane maintains a constant speed throughout the entire flight, which may not be the case due to factors such as wind and turbulence.

Learn more about speed here: https://brainly.com/question/27888149

#SPJ11

Please help with this question. I am offering 40 points. You can use the word box above to answer the questions.

Answers

We define [tex]9^{\frac{1}{2} }[/tex] to be the square root of nine. This means that [tex](9^{\frac{1}{2} })^{2}[/tex] must be equal to nine.

By using the properties of exponents to explain why [tex]8^{\frac{1}{3} }=2[/tex];

Statement                           Reasons_____________________

[tex]8^{\frac{1}{3} }=2[/tex]                                  Given

[tex](8^{\frac{1}{3} })^3=2^3[/tex]                        Exponent property of equality.

[tex]8^{\frac{1}{3} \times 3}=2^3[/tex]                        Exponent property of a power.

8 = 8                                simplify

What is an exponent?

In Mathematics, an exponent refers to a mathematical operation that is typically used in conjunction with an algebraic expression in order to raise a quantity to the power of another.

This ultimately implies that, an exponent is represented by the following mathematical expression;

bⁿ

Where:

the variables b and n are numerical values (numbers) or an algebraic expression.

n is referred to as a superscript or power.

By applying the multiplication and division law of exponents for powers to each of the expressions, we have the following:

[tex]9^{\frac{1}{2} }[/tex] = √9 (square root of nine)

[tex](9^{\frac{1}{2} })^{2}=(9^{\frac{1}{2} \times 2})=9^1 = 9[/tex]  

Read more on exponent here: https://brainly.com/question/27858496

#SPJ1

A student usally saves $20 a month. He would like to reach a goal of saving $350 in 12 months the students writes the equation 350=12(x + 20) to represent this situation

Answers

Answer: x=55/6 or 55 over 6

Step-by-step explanation: Step 1: Distribute:

- 350= 12(x+20)

- 350= 12x + 240

Step 2: Subtract 240 from both sides:

- 350-240= 12x+240-240

Step 3: Simplify:

Subtract the numbers: 350-240= 12x+240-240= 110=12x+240-240

Subtract again: 110=12x+240-240= 110=12x

Step 4: Divide both sides by the same factor:

110=12x= 110/12= 12x/12

Step 5: Simplify:

- Divide the numbers: 110/12=12x/12= x=55/6=12x/12

- Cancel terms that are in both the numerator and denominator: 55/6=12x/12= 55/6=x

- Move the variables to the left: 55/6=x = x=55/6

Answer: x=55/6 or 55 over 6

Calculate d²y/d²x y= -5x2 + x d²y/d²x= Calculate d²y/dx² y= 7/x d²y/dx²=

Answers

To calculate the second derivative of a function, we need to take the derivative of the first derivative. The second derivative gives us information about the curvature of the function. A positive second derivative indicates that the function is concave up, while a negative second derivative indicates that the function is concave down. A second derivative of zero indicates that the function has no curvature at that point.

In the first example given, y = -5x^2 + x, we first find the first derivative by taking the derivative of the function with respect to x. This gives us dy/dx = -10x + 1. To find the second derivative, we take the derivative of dy/dx with respect to x. This gives us d²y/d²x = -10. This indicates that the function has a constant negative curvature, meaning it is concave down everywhere.

In the second example given, y = 7/x, we first find the first derivative by taking the derivative of the function with respect to x. This gives us dy/dx = -7/x^2. To find the second derivative, we take the derivative of dy/dx with respect to x. This gives us d²y/dx² = 14/x^3. This indicates that the function is concave up for positive values of x and concave down for negative values of x. The second derivative is undefined at x = 0, indicating a point of inflection.

Overall, the second derivative gives us important information about the behavior of a function and can help us identify points of inflection and concavity.
MORE PROBLEM ON DERIVATIVES : https://brainly.com/question/23819325

#SPJ11

According the April 12, 2017 Pew Research survey, 58% of Americans approve of U. S. Missile strikes in


Syria in response to reports of the use of chemical weapons by Bashar al-Assad's government (the


Syrian government). A sample of 50 Americans are surveyed. Let o be the sample proportion of


Americans who approve the U. S. Missile strikes.


1. What is the population proportion?


(decimal form)


2. What is the sample size?


3. Can the normal approximation be used with this distribution?



4. What is the mean of the sampling proportion?

Answers

Answer:

The population proportion is given as 58% or 0.58 in decimal form.

The sample size is given as 50 Americans.

Yes, the normal approximation can be used with this distribution because the sample size is sufficiently large (n=50) and the underlying population is assumed to be large enough to satisfy the independence requirement.

The mean of the sampling proportion (o) can be calculated using the formula:

mean = population proportion = 0.58

Therefore, the mean of the sampling proportion is 0.58 or 58%.

To know more about decimal refer here

https://brainly.com/question/30958821#

#SPJ11

A tank in the shape of a hemisphere has a diameter of 8 feet. If the liquid that fills the tank has a density of 86 pounds per cubic foot, what is the total weight of the liquid in the tank, to the nearest full pound?

Answers

Answer:

209.07 pounds

Step-by-step explanation:

radius= 8÷2=4 feet

volume of hemisphere=((4/3)×(22/7)×r^3)/2

=134.09 cubic feet

Mass=density × volume

=86×134.09

=209.07 pounds

For each pair of numbers, decide if lines with these gradients are perpendicular or not. a) 5 and 1/ 5 b) 2/3 and -1/3 c) and -1/1 d) - and 3​

Answers

The pair of numbers  3/5 and -5/3,  -1/3 and 3 are perpendicular because two lines are perpendicular if and only if the product of their gradients is -1.

Each pair of numbers, we have to decide if lines with these gradients are perpendicular or not

Two lines are perpendicular if and only if the product of their gradients is -1.

For 5 and 1/5

The product is 1 which is not -1, so these are not perpendicular.

For 3/5 and -5/3

The product is -1 so these are perpendicular

For 1/4 and -1/4

The product is -1/16 so these are not perpendicular.

For -1/3 and 3

The product is  -1 so these are perpendicular

Hence, the pair of numbers  3/5 and -5/3,  -1/3 and 3 are perpendicular.

To learn more on slope of line click:

https://brainly.com/question/16180119

#SPJ1

Small baskets of tomatoes are sold at a vegetable stand for $3 per basket. Large


baskets of tomatoes are sold at the stand for $5 per basket. Only whole numbers of


baskets may be purchased.


A customer purchases a total of 8 baskets of tomatoes and pays $36.


A. Write and solve a system of equations that models the number of small


baskets (x) and the number of large baskets () that the customer purchases.


Show or explain all your work.

Answers

The customer purchased 2 small baskets and 6 large baskets.

Let x be the number of small baskets and y be the number of large baskets that the customer purchases.

We can set up a system of two equations based on the information given:

Equation 1: x + y = 8 (The total number of baskets purchased is 8)

Equation 2 3x + 5y = 36:  (The total amount paid for the baskets is $36)

To solve this system, we can use either substitution or elimination method.

Using substitution method:

From Equation 1, we have x = 8 - y.

Substitute this into Equation 2:

3(8 - y) + 5y = 36

24 - 3y + 5y = 36

2y = 12

y = 6

Now, we can substitute y = 6 back into Equation 1 to find x:

x + 6 = 8

x = 2

Therefore, the customer purchased 2 small baskets and 6 large baskets.

Using elimination method:

We can multiply Equation 1 by 3 and subtract it from Equation 2 to eliminate x:

 3x + 5y = 36

- (3x + 3y = 24)

   2y = 12

    y = 6

Now, we can substitute y = 6 back into either Equation 1 or Equation 2 to find x. Let's use Equation 1:

x + 6 = 8

x = 2

To know more about basket refer here

https://brainly.com/question/54060245#

#SPJ1

In the coordinate plane, the point X(1,4) is translated to the point X(0,6) . Under the same translation, the points Y(-1,2) and Z(-3,1) are translated to Y and Z , respectively. What are the coordinates of Y and Z ?
Help!!!!!! URGENT

Answers

Thus, the coordinates of Y and Z after the translation is obtained as :

Y'(-2,4) and Z'(-4, 3).

Define about the translation:

A figure is translated when it is moved from one point to another without changing in size, form, or rotation.

A figure can be translated to move it up, down, left, or right while maintaining the same size. This is carried out using a coordinate system in order to be done properly and accurately.The pre-image is the original object that needs to be translated, and the image is the translated object.

Given translation:

Point X(1,4) --->  point X'(0,6)

There is 1 unit shift to left as 1 is subtracted to x coordinate to get 0.

There is 2 unit shift to upward as 2 is added to y coordinate to get 6..

Translation:

(x,y) --->(x - 1, y + 2)

Applying same on points Y and Z,

Y(-1,2)  --> Y'(-2,4)

Z(-3,1)  --> Z'(-4, 3)

Thus, the coordinates of Y and Z after the translation is obtained as :

Y'(-2,4) and Z'(-4, 3).

Know more about the translation

https://brainly.com/question/1574635

#SPJ1

Other Questions
do not tell a lie. tell me active voice Calculate the amount of electrical energy (in kWh) needed to produce1.00E3 kg of aluminum using electrolysis if the applied voltage is 6.00 V. (1kWh = 3.6E6 J) How does free trade impact what you do or do not buy? Explain in a full paragraph of 3+ sentences.(Economics question) Piscine is replacing the paving stones around her inground pool. Her pool is 10 mby 5 m, and is surrounded by a 1. 5 m border of paving stones. a) How many square metres of paving stones will she need in total?b) If each paving stone is 25 cm by 40 cm, in theory, how many paving stoneswill she need?c) Will your answer in part b) actually be enough? Try fitting the stones in thespace to see whether Piscine can complete the border with exactly thatnumber of stones, or whether there will be waste, requiring some extras. Paige walks to the park 2/3 mile away it takes her 16 minutes to get there how many miles per minutes Committee hearings can occur in the Texas State House and the Texas State SenateTrueFalse how much soda do you need if you buy 20 bags of 4 bags of hot chips and 3 bottles of soda. under a time crunch, you only have time to take a sample of 15 water bottles and measure their contents. the sample had a mean of 20.05 ounces with a sample standard deviation of 0.3 ounces. what would be the 90% confidence interval, when we assumed these measurements are normally distributed? g Georgian entrepreneur helped Georgias economy grow by turning an invention for a new type of hosiery it into a multibillion dollar industry which of the following organization structures is usually bureaucratic, the division of labor is typically well defined, and is suitable for companies in which processes are routine and efficiency is of primary importance? a. decentralized b. tall c. centralized d. flat e. hierarchical the demand curve facing a firm in monopolistic competition is downward sloping, because the firm group of answer choices sells a differentiated product. is the entire industry by itself. is big relative to the market. none of the above Did Jackson ultimately preserve or limit the spread of liberty in America?1-Respond to the question. Take a position in agreement or disagreement. 2-Explain why you took that position and give 3 points of evidence in support of your position. 3-Provide 3 sources of evidence that support your position by copying and pasting a web link to each. Thank you!!! no links they dont work:) [tex]-2-\frac{5}{4} x=13\\[/tex] Social Participation is the backbone of socialization Justify the Statement? 3.2.6 Quiz: The Rise and Fall of RomeWhich of these statements best compares the Roman Republic with ancientGreece's direct democracy?OA. The Roman concept of appointing a dictator to rule during timesof crisis came directly from Greek direct democracy. So theRoman Republic had no voters.OB. In both forms of government, voters elected representatives.Those representatives were the only people who could vote onspecific laws.OC. Voters elected representatives in the Roman Republic. In theGreek direct democracy, voters themselves enacted laws andpolicies.D. Greek direct democracy was a form of monarchy, so there were novoters In the Roman Republic voters elected representatives whoEnacted law and policies HELPPPP !!Mr. Olaffsen opened a sandwich shop and a smoothie stand in his neighborhood.The following table and equation show function f, representing Mr. Olaffsen's profit, in dollars, x months since opening the sandwich shop.__________________________________________________________________x 1 2 3 4 5 6 7f(x) 12,000 15,500 18,000 19,500 20,000 19,500 18,000__________________________________________________________________The following table and equation show function g, representing Mr. Olaffsen's profit, in dollars, x months since opening the smoothie stand.__________________________________________________________________x 1 2 3 4 5 6 7g(x) 9,300 12,000 14,100 15,600 16,500 16,800 16,500__________________________________________________________________Select the true statement. A.) The difference between the maximum profit earned by the sandwich shop and the smoothie stand is $3,000. B.) The difference between the maximum profit earned by the sandwich shop and the smoothie stand is $3,500. C. ) The difference between the maximum profit earned by the sandwich shop and the smoothie stand is $3,200. D. ) The difference between the maximum profit earned by the sandwich shop and the smoothie stand is $2,700. Ankush became popular. His social work. And For Because Though Escriba la forma indicada de haber.Summary of Present PerfectIndicative: he, has, ha, hemos, habis, han + past participleSubjunctive: haya, hayas, haya, hayamos, hayis, hayan + past participlePlease be right ratios of sin y and cos x share? What does lightning have in common with the shock you receive when you touch a doorknob?.